2014 dxdy logo

Научный форум dxdy

Математика, Физика, Computer Science, Machine Learning, LaTeX, Механика и Техника, Химия,
Биология и Медицина, Экономика и Финансовая Математика, Гуманитарные науки


Правила форума


В этом разделе нельзя создавать новые темы.

Если Вы хотите задать новый вопрос, то не дописывайте его в существующую тему, а создайте новую в корневом разделе "Помогите решить/разобраться (М)".

Если Вы зададите новый вопрос в существующей теме, то в случае нарушения оформления или других правил форума Ваше сообщение и все ответы на него могут быть удалены без предупреждения.

Не ищите на этом форуме халяву, правила запрещают участникам публиковать готовые решения стандартных учебных задач. Автор вопроса обязан привести свои попытки решения и указать конкретные затруднения.

Обязательно просмотрите тему Правила данного раздела, иначе Ваша тема может быть удалена или перемещена в Карантин, а Вы так и не узнаете, почему.



Начать новую тему Ответить на тему На страницу 1, 2  След.
 
 Наименьшее значение выражения
Сообщение26.02.2009, 15:24 


23/02/09
15
Минск
Дана система:

$\left\{ \begin{array}{l}A=\left(a+\frac{1}{a}\right)^2 + \left(b+\frac{1}{b}\right)^2 \\ a+b=1 \\ a>0,  b>0 \end{array} \right $

Найти наименьшее значение, которое может принимать выражение $A$.
Мой ответ $A_{min}=12,5$
Хотелось бы узнать ваши способы решения и ответы.

 Профиль  
                  
 
 
Сообщение26.02.2009, 15:46 
Заслуженный участник
Аватара пользователя


13/08/08
14457
А выразить $b$ через $a$, подставить и продифференцировать?
А еще проще - обе скобки принимают наименьшее значение в одной и той же точке.
А посложнее $a=0.5-t; \quad b=0.5+t$
Ещё покруче - условным экстремумом через множитель Лагранжа.

 Профиль  
                  
 
 
Сообщение26.02.2009, 15:55 
Заслуженный участник
Аватара пользователя


30/01/09
6686
Можно решать через функцию Лагранжа. Получается что минимиум достигается на середине отрезка. Так что ответ Ваш верен.

 Профиль  
                  
 
 
Сообщение26.02.2009, 16:08 
Заслуженный участник


11/05/08
32166
Можно вообще практически не решать. Функция, очевидно, выпукла, и её единственный глобальный экстремум (1;1) тоже очевиден и находится за пределами области. Горизонтальная и вертикальная стенки отпадают -- там функция уходит на бесконечность. Остаётся только наклонная стенка. А там и вовсе ничего не надо считать -- в силу симметрии задачи и опять же выпуклости минимум может быть только в её середине, т.е. в точке (0.5;0.5).

 Профиль  
                  
 
 
Сообщение26.02.2009, 16:12 
Заблокирован
Аватара пользователя


16/12/08

467
Краснодар
MathCad выдал три минимума:
$a=-0.75487$
$$a=\frac 12$$
$a=1.75487$
Под заданные ограничения подпадает только $$a=\frac 12$$. Для него $A=12,5$

 Профиль  
                  
 
 
Сообщение26.02.2009, 16:35 
Заслуженный участник
Аватара пользователя


13/08/08
14457
а почему тема называется "Наибольшее значение выражения", а мы ищем наименьшее? Нет ли здесь какого-либо покушения на основы?

 Профиль  
                  
 
 
Сообщение26.02.2009, 16:37 
Заслуженный участник


11/05/08
32166
А это потому, что тема -- дискуссионная. Вот вам и повод для дискуссии.

 Профиль  
                  
 
 
Сообщение26.02.2009, 16:44 


23/02/09
15
Минск
Мое решение чисто алгебраическое :)

Существует лемма: $a^2 + b^2 \geqslant \frac{(a+b)^2}{2}$
Тогда справедливо неравенство

$\left(a+\frac{1}{a}\right)^2 + \left(b+\frac{1}{b}\right)^2 \geqslant \frac{(a+1/a+b+1/b)^2}{2}$
$\left(a+\frac{1}{a}\right)^2 + \left(b+\frac{1}{b}\right)^2 \geqslant \frac{(1+\frac{1}{b-b^2})^2}{2}=B$

Чтобы найти $A_{min}$ надо найти $B_{min}$, а для этого надо найти $max$ для $b-b^2$

Рассмотрим функцию $f(b)=-b^2+b$

$f_{max}(b)=f(b_0)=|b_0=\frac{1}{2}|=\frac{1}{4}$
Т.е. $max(b-b^2)=\frac{1}{4}$. Тогда $min\left(\frac{1}{b-b^2}\right)=4$

Подставляем в выражение $B=\frac{(1+\frac{1}{b-b^2})^2}{2}=\frac{(1+4)^2}{2}=12,5$

Расписал как можно подробнее :)

Добавлено спустя 1 минуту 8 секунд:

gris писал(а):
а почему тема называется "Наибольшее значение выражения", а мы ищем наименьшее? Нет ли здесь какого-либо покушения на основы?

Извиняюсь, сейчас исправлю :)

 Профиль  
                  
 
 
Сообщение26.02.2009, 17:01 
Заслуженный участник


09/02/06
4382
Москва
Проще всего воспользоваться выпуклостью функции $f(x)=(x+\frac 1x )^2$.
Тогда $f(a)+f(b)\ge 2f(\frac{a+b}{2})=12.5$ как только $a+b=1$.

 Профиль  
                  
 
 
Сообщение26.02.2009, 17:03 
Заслуженный участник


11/05/08
32166
Georgise писал(а):
$\left(a+\frac{1}{a}\right)^2 + \left(b+\frac{1}{b}\right)^2 \geqslant \frac{(1+\frac{1}{b-b^2})^2}{2}=B$

Во-первых, это непонятно откуда взято, а во-вторых -- и неверно. Подставьте $b\approx1$.

 Профиль  
                  
 
 
Сообщение26.02.2009, 21:03 


23/02/09
15
Минск
ewert писал(а):
Georgise писал(а):
$\left(a+\frac{1}{a}\right)^2 + \left(b+\frac{1}{b}\right)^2 \geqslant \frac{(1+\frac{1}{b-b^2})^2}{2}=B$

Во-первых, это непонятно откуда взято, а во-вторых -- и неверно. Подставьте $b\approx1$.


$\left(a+\frac{1}{a}\right)^2 + \left(b+\frac{1}{b}\right)^2 \geqslant \frac{(a+\frac{1}{a}+b+\frac{1}{b})^2}{2}$ - это исходя из того, что неравенство

$a^2 + b^2 \geqslant \frac{(a+b)^2}{2}$ верно при любых $a$ и $b$.

Ну а приведя числитель $(a+\frac{1}{a}+b+\frac{1}{b})^2$ к общему знаменателю и получаем $(1+\frac{1}{b-b^2})^2$

Так что все верно на мой взгляд. И если подставить $b\approx1$ то неравенство выполнится, я проверил :)

 Профиль  
                  
 
 
Сообщение26.02.2009, 21:10 
Заслуженный участник


11/05/08
32166
ну, во-первых, не получаем, а во-вторых, почему Вы решили, что константа плюс два всегда больше плюс бесконечности?...

 Профиль  
                  
 
 
Сообщение26.02.2009, 21:14 


23/02/09
15
Минск
ewert писал(а):
ну, во-первых, не получаем, а во-вторых, почему Вы решили, что константа плюс два всегда больше плюс бесконечности?...


Покажите, где у меня написано что-нибудь наподобие этого, пожалуйста.

 Профиль  
                  
 
 
Сообщение26.02.2009, 21:34 
Заслуженный участник


11/05/08
32166
Ну это ж Вы написали, не так ли?

Georgise писал(а):
$\left(a+\frac{1}{a}\right)^2 + \left(b+\frac{1}{b}\right)^2 \geqslant \frac{(1+\frac{1}{b-b^2})^2}{2}=B$


При $b=1$ получаем слева $(a+{1\over a})^2+2$, а справа -- бесконечность, которая якобы меньше левой части.

 Профиль  
                  
 
 
Сообщение26.02.2009, 21:51 


23/02/09
15
Минск
ewert писал(а):
При $b=1$ получаем слева $(a+{1\over a})^2+2$, а справа -- бесконечность, которая якобы меньше левой части.


Уважаемый:)
В условии есть такая строчка:
$\left\{ \begin{array}{l}a+b=1 \\ a>0,  b>0 \end{array} \right $

Это означает, что область допустимых значений для $a$ и $b$:
$\left\{ \begin{array}{l}0<a<1 \\ 0<b<1\end{array} \right $

Т.е. $b\neq1$ по условию.
А вот если подставить $b\approx1$, например $b=0,(9)$ или просто $b=0,9$, то все получится.

 Профиль  
                  
Показать сообщения за:  Поле сортировки  
Начать новую тему Ответить на тему  [ Сообщений: 18 ]  На страницу 1, 2  След.

Модераторы: Модераторы Математики, Супермодераторы



Кто сейчас на конференции

Сейчас этот форум просматривают: нет зарегистрированных пользователей


Вы не можете начинать темы
Вы не можете отвечать на сообщения
Вы не можете редактировать свои сообщения
Вы не можете удалять свои сообщения
Вы не можете добавлять вложения

Найти:
Powered by phpBB © 2000, 2002, 2005, 2007 phpBB Group